How Do You Solve This Arithmetico-Geometric Series?

Click For Summary
The discussion focuses on solving the infinite series \(\sum \frac{n+1}{4^{n}}\) and identifying it as an arithmetico-geometric series. Participants suggest various methods, including breaking the series into separate sums and using integration techniques to find a closed form. One approach involves multiplying the series terms by \(x^n\), integrating, and then differentiating to derive the original sum. The conversation emphasizes the challenge of applying standard geometric series solutions to this specific series type. Ultimately, the goal is to compute the sum effectively.
eNathan
Messages
351
Reaction score
2
Trying to solve this infinite series??

Hey folks! I've spent hours trying to solve this and have exhausted all available resources.. I just need to be pointed in the right direction!

Homework Statement


Compute the sum of the infinite series (I believe this is an arithmetico geometric series):
\sum \frac{n+1}{4^{n}}

For n=0..infinity

Homework Equations


I know the standard way to solve a geometric series, but don't know how to solve this type of series.


The Attempt at a Solution


I've set up something like this:
S_{n} = \sum \frac{n+1}{4^{n}}
I've tried multiplying by 1/4, 4 and other logical things, but am just not sure how to proceed.

Thanks in advance!
 
Physics news on Phys.org
Your expression can be written as
$$\sum_0^\infty \frac{1}{4^{n}} + \sum_1^\infty \frac{1}{4^{n}} + \sum_2^\infty \frac{1}{4^{n}} + \dots$$
(This assumes you start your sum at 0, if you start at 1 you have to modify it a bit.)
 
Another approach, perhaps more general in application, is to multiply the terms by xn then integrate. You should then be able to sum the series into closed form and differentiate to get a closed form for the original sum. Finally set x=1.
 
Question: A clock's minute hand has length 4 and its hour hand has length 3. What is the distance between the tips at the moment when it is increasing most rapidly?(Putnam Exam Question) Answer: Making assumption that both the hands moves at constant angular velocities, the answer is ## \sqrt{7} .## But don't you think this assumption is somewhat doubtful and wrong?

Similar threads

  • · Replies 1 ·
Replies
1
Views
2K
  • · Replies 7 ·
Replies
7
Views
3K
Replies
4
Views
2K
  • · Replies 7 ·
Replies
7
Views
2K
  • · Replies 2 ·
Replies
2
Views
1K
  • · Replies 3 ·
Replies
3
Views
2K
  • · Replies 17 ·
Replies
17
Views
3K
  • · Replies 14 ·
Replies
14
Views
2K
Replies
2
Views
1K
  • · Replies 9 ·
Replies
9
Views
3K